subject
Mathematics, 08.05.2021 09:00 Carlosanddana123

Verify the following reduction formula: \displaystyle \int \sec^n(u)\, du=\frac{\sec^{n-2}(u)\tan(u)}{n-1}+\frac{n-2}{n-1}\int \sec^{n-2}(u)\, du, \; n\neq 1

ansver
Answers: 2

Another question on Mathematics

question
Mathematics, 21.06.2019 13:30
Find the magnitude of the torque about p if an f = 80-lb force is applied as shown. (round your answer to the nearest whole number.) ft-lb
Answers: 1
question
Mathematics, 21.06.2019 14:30
Our goal is to collect email addresses from 60% of customers who purchase a product." so that means if i talk to 95 customers, i need to persuade to provide an email address?
Answers: 1
question
Mathematics, 21.06.2019 23:00
Astore sells 4 apples for $3 and 3 oranges for $4 if pete buys 12 apples and 12 oranges how much will it cost
Answers: 1
question
Mathematics, 21.06.2019 23:30
Side xy of triangle xyz is extended to point w, creating a linear pair with ∠wyz and ∠xyz. measure of an exterior angle what is the value of x? 64 80 100 180
Answers: 2
You know the right answer?
Verify the following reduction formula: ​...
Questions
question
Mathematics, 28.01.2021 20:20
question
Mathematics, 28.01.2021 20:20
question
Mathematics, 28.01.2021 20:20
question
Mathematics, 28.01.2021 20:20
Questions on the website: 13722367